Anda di halaman 1dari 39

EE 369

POWER SYSTEM ANALYSIS

Lecture 16
Economic Dispatch
Tom Overbye and Ross Baldick

1
Announcements
Read Chapters 6 (section 6.12) and 7
(sections 7.1 to 7.3).
Homework 12 is 6.62, 6.63, 6.67 (calculate
economic dispatch for values of load from
55 MW to 350 MW); due Tuesday, 11/29.
Class review and course evaluation on
Tuesday, 11/29.
Midterm III on Thursday, 12/1, including
material through Homework 12.
2
Economic Dispatch: Formulation
The goal of economic dispatch is to
determine the generation dispatch that
minimizes the instantaneous operating cost,
subject to the constraint that total
generation = total load + losses
m
Minimize CT Ci ( PGi )
i 1 Initially we'll
Such that ignore generator
m limits and the
PGi PD PLosses losses
i 1 3
Unconstrained Minimization
This is a minimization problem with a single
equality constraint
For an unconstrained minimization a
necessary (but not sufficient) condition for a
minimum is the gradient of the function
must be zero, f (x) 0
The gradient generalizes the first derivative
for multi-variable problems:
f (x) f (x) f (x)
f (x) x , x , ,
1 2 xn
4
Minimization with Equality Constraint
When the minimization is constrained with an
equality constraint we can solve the problem
using the method of Lagrange Multipliers
Key idea is to represent a constrained
minimization problem as an unconstrained
problem.
That is, for the general problem
minimize f (x) s.t. g(x) 0
We define the Lagrangian L(x, ) f (x) T g (x)
Then a necessary condition for a minimum is the
L x (x, ) 0 and L (x, ) 0 5
Economic Dispatch Lagrangian
For the economic dispatch we have a minimization
constrained with a single equality constraint
m m
L(PG , ) Ci ( PGi ) ( PD PGi ) (no losses)
i 1 i 1
The necessary conditions for a minimum are
L(PG , ) dCi ( PGi )
0 (for i 1 to m)
PGi dPGi
m
PD PGi 0
i 1
6
Economic Dispatch Example
What is economic dispatch for a two generator
system PD PG1 PG 2 500 MW and
C1 ( PG1 ) 1000 20 PG1 0.01PG21 $/h
C2 ( PG 2 ) 400 15PG 2 0.03PG22 $/h
Using the Lagrange multiplier method we know:
dC1 ( PG1 )
20 0.02 PG1 0
dPG1
dC2 ( PG 2 )
15 0.06 PG 2 0
dPG 2
500 PG1 PG 2 0
7
Economic Dispatch Example, contd
We therefore need to solve three linear equations
20 0.02 PG1 0
15 0.06 PG 2 0
500 PG1 PG 2 0
0.02 0 1 PG1 20
0 0.06 1 PG 2 15

1 1 0 500
PG1 312.5 MW
P 187.5 MW
G2
26.2 $/MWh 8
Economic dispatch example, contd
At the solution, both generators have the
same marginal (or incremental) cost, and this
common marginal cost is equal to .
Intuition behind solution:
If marginal costs of generators were different,
then by decreasing production at higher marginal
cost generator, and increasing production at lower
marginal cost generator we could lower overall
costs.
Generalizes to any number of generators.
If demand changes, then change in total costs
can be estimated from . 9
Economic dispatch example, contd
Another way to solve the equations is to:
Rearrange the first two equations to solve for PG1
and PG2 in terms of ,
Plug into third equation and solve for ,
Use the solved value of to evaluate PG1 and PG2.
This works even when relationship between
generation levels and is more complicated:
Equations are more complicated than linear when
there are maximum and minimum generation
limits or we consider losses. 10
Lambda-Iteration Solution Method
Discussion on previous page leads to lambda-
iteration method:
this method requires a unique mapping from a value
of lambda (marginal cost) to each generators MW
output: PGi().
for any choice of lambda (common marginal cost),
the generators collectively produce a total MW
output,
the method then starts with values of lambda below
and above the optimal value (corresponding to too
little and too much total output), and then
iteratively brackets the optimal value. 11
Lambda-Iteration Algorithm
Pick L and H such that
m m
Gi ) PD 0
P ( L
Gi ) PD 0
P ( H

i 1 i 1

While H L Do
M ( H L ) / 2
m
If Gi
P ( M
) PD 0 Then H
M

i 1

Else L M
End While 12
Lambda-Iteration: Graphical View
In the graph shown below for each value of lambda
there is a unique PGi for each generator. This
relationship is the PGi() function.

13
Lambda-Iteration Example
Consider a three generator system with
IC1 ( PG1 ) 15 0.02 PG1 $/MWh
IC2 ( PG 2 ) 20 0.01PG 2 $/MWh
IC3 ( PG 3 ) 18 0.025 PG 3 $/MWh
and with constraint PG1 PG 2 PG 3 1000MW
Rewriting generation as a function of , PGi ( ),
we have
15 20
PG1 ( ) PG2 ( )
0.02 0.01
18
PG3 ( ) 14
0.025
Lambda-Iteration Example, contd
m
Pick L so Gi ) 1000 0 and
P ( L

i=1
m
Gi ) 1000 0
P ( H

i=1
m
Try L 20 then PGi (20) 1000
i 1
15 20 18
1000 670 MW
0.02 0.01 0.025
m
Try H 30 then PGi (30) 1000 1230 MW
i 1
15
Lambda-Iteration Example, contd
Pick convergence tolerance 0.05 $/MWh
Then iterate since H L 0.05

M ( H L ) / 2 25
m
Then since Gi
P (25) 1000 280 we set H
25
i 1
Since 25 20 0.05
M (25 20) / 2 22.5
m
Gi
P (22.5) 1000 195 we set L
22.5
i 1 16
Lambda-Iteration Example, contd
Continue iterating until H L 0.05

The solution value of , , is 23.53 $/MWh


*

Once * is known we can calculate the PGi


23.53 15
PG1 (23.5) 426 MW
0.02
23.53 20
PG 2 (23.5) 353 MW
0.01
23.53 18
PG 3 (23.5) 221 MW
0.025
17
Thirty Bus ED Example
Case is economically dispatched (without considering
the incremental impact of the system losses).

18
Generator MW Limits
Generators have limits on the minimum and
maximum amount of power they can
produce
Typically the minimum limit is not zero.
Because of varying system economics usually
many generators in a system are operated at
their maximum MW limits:
Baseload generators are at their maximum limits
except during the off-peak.
19
Lambda-Iteration with Gen Limits

In the lambda-iteration method the limits are taken


into account when calculating PGi ( ) :
if calculated production for PGi PGi ,max
then set PGi ( ) PGi ,max
if calculated production for PGi PGi ,min
then set PGi ( ) PGi ,min

20
Lambda-Iteration Gen Limit Example
In the previous three generator example assume
the same cost characteristics but also with limits
0 PG1 300 MW 100 PG2 500 MW
200 PG3 600 MW
With limits we get:
m
PGi (20) 1000 PG1 (20) PG 2 (20) PG 3 (20) 1000
i 1
250 100 200 1000
450 MW (compared to 670MW)
m
PGi (30) 1000 300 500 480 1000 280 MW
i 1 21
Lambda-Iteration Limit Example,contd
Again we continue iterating until the convergence
condition is satisfied.
With limits the final solution of , is 24.43 $/MWh
(compared to 23.53 $/MWh without limits).
Maximum limits will always cause to either increase
or remain the same.
Final solution is:
PG1 (24.43) 300 MW (at maximum limit)
PG 2 (24.43) 443 MW
PG 3 (24.43) 257 MW
22
Back of Envelope Values
$/MWhr = fuelcost * heatrate + variable O&M
Typical incremental costs can be roughly
approximated:
Typical heatrate for a coal plant is 10, modern
combustion turbine is 10, combined cycle plant is
6 to 8, older combustion turbine 15.
Fuel costs ($/MBtu) are quite variable, with
current values around 2 for coal, 3 to 5 for natural
gas, 0.5 for nuclear, probably 10 for fuel oil.
Hydro costs tend to be quite low, but are fuel
(water) constrained
Wind and solar costs are zero. 23
Inclusion of Transmission Losses
The losses on the transmission system are a
function of the generation dispatch.
In general, using generators closer to the
load results in lower losses
This impact on losses should be included
when doing the economic dispatch
Losses can be included by slightly rewriting
the Lagrangian to include losses PL:
m m
L(PG , ) Ci ( PGi ) PD PL ( PG ) PGi
i 1 i 1 24
Impact of Transmission Losses
The inclusion of losses then impacts the necessary
conditions for an optimal economic dispatch:
m m
L(PG , ) Ci ( PGi ) PD PL ( PG ) PGi .
i 1 i 1
The necessary conditions for a minimum are now:
L(PG , ) dCi ( PGi ) PL ( PG )
1 0
PGi dPGi PGi
m
PD PL ( PG ) PGi 0
i 1
25
Impact of Transmission Losses
dCi ( PGi ) PL ( PG )
Solving for , we get: 1 0
dPGi PGi
1 dCi ( PGi )

PL ( PG ) dPGi
1 P
Gi

Define the penalty factor Li for the i th generator


(don't confuse with Lagrangian L!!!)
1 The penalty factor
Li
PL ( PG ) at the slack bus is
1 P always unity!
Gi 26
Impact of Transmission Losses
The condition for optimal dispatch with losses is then
L1 IC1 ( PG1 ) L2 IC2 ( PG 2 ) Lm ICm ( PGm )
1
Li . So, if increasing PGi increases
PL ( PG )
1 P
Gi
PL ( PG )
the losses then 0 Li 1.0
PGi
This makes generator i appear to be more expensive
(i.e., it is penalized). Likewise Li 1.0 makes a generator
appear less expensive.
27
Calculation of Penalty Factors
Unfortunately, the analytic calculation of Li is
somewhat involved. The problem is a small change
in the generation at PGi impacts the flows and hence
the losses throughout the entire system. However,
using a power flow you can approximate this function
by making a small change to PGi and then seeing how
the losses change:
PL ( PG ) PL ( PG ) 1
Li
PGi PGi PL ( PG )
1
PGi 28
Two Bus Penalty Factor Example

PL ( PG ) PL ( PG ) 0.37 MW
0.0387 0.037
PG 2 PG 2 10 MW
L2 0.9627 L2 0.9643 29
Thirty Bus ED Example
Now consider losses.
Because of the penalty factors the generator incremental
costs are no longer identical.

30
Area Supply Curve
The area supply curve shows the cost to produce the
next MW of electricity, assuming area is economically
dispatched 10.00

7.50

Supply
curve for
5.00
thirty bus
system
2.50

0.00
0 100 200 300 400
Total Area Generation (MW)

31
Economic Dispatch - Summary
Economic dispatch determines the best way to
minimize the current generator operating costs.
The lambda-iteration method is a good approach for
solving the economic dispatch problem:
generator limits are easily handled,
penalty factors are used to consider the impact of losses.
Economic dispatch is not concerned with determining
which units to turn on/off (this is the unit commitment
problem).
Basic form of economic dispatch ignores the
transmission system limitations.
32
Security Constrained ED
or Optimal Power Flow
Transmission constraints often limit ability to use
lower cost power.
Such limits require deviations from what would
otherwise be minimum cost dispatch in order to
maintain system security.
Need to solve or approximate power flow in
order to consider transmission constraints.

33
Security Constrained ED
or Optimal Power Flow
The goal of a security constrained ED or
optimal power flow (OPF) is to determine the
best way to instantaneously operate a
power system, considering transmission limits.
Usually best = minimizing operating cost,
while keeping flows on transmission below
limits.
In three bus case the generation at bus 3 must
be limited to avoid overloading the line from
bus 3 to bus 2. 34
Security Constrained Dispatch
-22 MW 22 MW
Bus 2 4 MVR -4 MVR Bus 1
1.00 PU
357 MW
179 MVR
1.00 PU
0 MW
100% 37 MVR
-122 MW
194 MW OFF AGC -142 MW
49 MVR 41 MVR 100 MW
232 MVR AVR ON
145 MW 100% 124 MW
-37 MVR -33 MVR
Bus 3 1.00 PU
Home Area
179 MW
Scheduled Transactions 89 MVR
100.0 MW 448 MW AGC ON
19 MVR AVR ON

Need to dispatch to keep line


from bus 3 to bus 2 from overloading
35
Multi-Area Operation
In multi-area system, rules have been established
regarding transactions on tie-lines:
In Eastern interconnection, in principle, up to nominal
thermal interconnection capacity,
In Western interconnection there are more complicated
rules
The actual power that flows through the entire network
depends on the impedance of the transmission lines,
and ultimately determine what are acceptable patterns
of dispatch:
Can result in need to curtail transactions that otherwise
satisfy rules.
Economically uncompensated flow through other areas
is known as parallel path or loop flows.
Since ERCOT is one area, all of the flows on AC lines are
inside ERCOT and there is no uncompensated flow on
AC lines. 36
Seven Bus Case: One-line
System has
three areas 44 MW -42 MW -31 MW 31 MW 80 MW
1.05 PU
3
0.99 PU
4
30 MVR Top area
1 1.00 PU
62 MW 106 MW -37 MW 110 MW -32 MW
has five
AGC ON 40 MVR
94 MW
buses
Case Hourly Cost AGC ON
38 MW 16933 $/MWH
-14 MW
-61 MW
32 MW
1.04 PU 79 MW -77 MW 1.01 PU
2 Top Area Cost 5
8029 $/MWH
40 MW
40 MW -39 MW 130 MW No net
20 MVR 40 MVR
168 MW AGC ON interchange
-40 MW 20 MW -20 MW
40 MW between
1.04 PU 1.04 PU Any areas.
6 20 MW 7
-20 MW
200 MW 200 MW
0 MVR Left Area Cost Right Area Cost
Left area 200 MW AGC ON
4189 $/MWH 4715 $/MWH 0 MVR
201 MW AGC ON
has one
Right area has one
bus
bus 37
Seven Bus Case: Area View
Area Losses
Top 7.09 MW

Actual
40.1 MW
flow
-40.1 MW
0.0 MW
0.0 MW between
System has areas
40 MW of
Scheduled
Loop Flow
Left Right
flow
Area Losses 40.1 MW Area Losses
0.33 MW 0.0 MW 0.65 MW

Loop flow can result in higher losses


38
Seven Bus - Loop Flow?
Area Losses
Top 9.44 MW

Note that
4.8 MW
0.0 MW
-4.8 MW Tops
0.0 MW
Losses have
increased
from
7.09MW to
Left Right
9.44 MW
Area Losses 104.8 MW Area Losses
-0.00 MW 100.0 MW 4.34 MW

100 MW Transaction Transaction has


between Left and Right actually decreased
the loop flow 39

Anda mungkin juga menyukai